Pediatrics AAP Board Review Questions

Ace your homework & exams now with Quizwiz!

What does initial evaluation of decreased level of consciousness include?

Serum chemistries, toxicology, infectious evaluation, or central nervous system imaging based on the history and physical examination.

What are the ARDS gas goals to achieve?

Aterial PO2 levels of 60 mm Hg or higher, and allowing higher PaCO2 levels to limit inflation pressures as long as the pH is greater than 7.25.

What is the definition of a one-eyed athlete?

Athletes are considered functionally 1-eyed if they have corrected vision of worse than 20/40 in 1 eye.

What are the four ethical principles

Autonomy: Allowing or enabling patients to make their own informed decisions about which health care interventions they will or will not receive Beneficence: The obligation to act for the benefit of others; to do good Nonmaleficence: The obligation to minimize or eliminate harm Justice: The obligation to treat others equally and to distribute benefits and burdens fairly

Pt with isolated HA, vomiting, and scalp hematomas are intermediate category. What is the next best step in management?

The PECARN guidelines recommend that the decision to perform brain CT versus clinical observation only should be made based on additional clinical factors, such as the presence of single versus multiple intermediate risk factors, age younger than 3 months, worsening symptoms or signs after ED observation, physician experience, and parental preference.

How does PECARN tie into brain CT utilization?

The PECARN study investigators found that brain CT utilization was lower among children who were observed following closed head trauma without a higher rate of clinically significant traumatic brain injuries, especially if their symptoms improved during the observation period.

Stickler Syndrome

most common identifiable cause of cleft palate (57%), 1/7500-9000 births, autosomal dominant. Characteristics include cleft palate, SNHL, progressive myopia, retinal detachent and cataracts, progressive arthropathy (joint disease). Craniofacialfeatures include maxillary and mandibular hypoplasia, flat midface impression, prominant eyes, long philtrum, depressed nasal bridge with epicanthal folds, cleft palate. Co-occur with pierre robin sequence

Stickler syndrome

most common identifiable cause of cleft palate (57%), 1/7500-9000 births, autosomal dominant. Characteristics include cleft palate, SNHL, progressive myopia, retinal detachent and cataracts, progressive arthropathy (joint disease). Craniofacialfeatures include maxillary and mandibular hypoplasia, flat midface impression, prominant eyes, long philtrum, depressed nasal bridge with epicanthal folds, cleft palate. Co-occur with pierre robin sequence

What type of anterior mediastinal masses can cause SVC?

non-Hodgkin lymphoma, acute lymphocytic leukemia, and Hodgkin disease.

A 17-year-old, sexually active boy has complaints of intermittent burning with urination for the last 2 weeks. What is the likely diagnosis?

nongonococcal urethritis

What are other criteria besides cafe-au-lait spots that are needed in the diagnosis of NF-1?

optic nerve glioma, Lisch nodules, sphenoid bone dysplasia, a bony pseudarthrosis, or a first-degree relative with NF1.

Children usually get retrophyarngeal abscesses vs. PTA. What age?

peritonsillar abscesses are most common in adolescents and young adults.

What is the definition of microalbuminuria?

range of 30 to 330 mg/day

What is the definition of proteinuria?

range of 330 mg/day or more. Urine dipsticks become positive when urine albumin secretion is more than 300 mg/day.

Kostmann Syndrome - severe congenital neutropenia

rare inherited disorder of neutrophil production, usually detected soon after birth

What is the USPTF reccommendation for infants found to have sickle cell anemia?

receive prophylactic penicillin (125 mg by mouth twice daily) by 2 months of age and receive pneumococcal vaccines at recommended intervals.

What is sickle cell trait associated with?

rhabdomyolysis after extreme physical exertion, hyphema and glaucoma after eye trauma, renal dysfunction, and thrombosis; however, these associations are not yet definitive.

When is the maternal quad screen utilized?

second trimester of pregnancy between the 15th and 20th week to assess the risk that a fetus may have a chromosomal abnormality such as trisomy 21, trisomy 18, or an open neural tube defect/anencephaly.

Adolescent with 2-month history of fevers, anorexia, and fatigue. and facial and upper extremity swelling, jugular venous distention, headache, and air hunger. What is the likely diagnosis?

superior vena cava (SVC) syndrome, which can occur from lymphoma with an anterior mediastinal mass.

A girl who has a high piercing of her ear now presents with ear pain. What is the likely concern?

suppurative auricular perichondritis.

What are the other mediastinal masses but are less likely to cause SVC syndrome.

teratoma, thyroid carcinoma, and enlargement of the thymus

You are seeing an 18-month-old boy who has had 2 skin abscesses in the perianal region and 1 abscess in the scalp, recurrent oral ulcers, 5 episodes of otitis media, and 2 episodes of pneumonia in the previous year. MOST appropriate next steps in the management?

testing for the phagocyte oxidative burst by flow cytometry and starting prophylaxis with trimethoprim and sulfamethoxazole CGD= The X-linked form is by far the most common and is caused by a mutation in the CYBB gene that results in a dysfunctional gp91 protein.

What are criteria for inpatient admission for bacterial pneumonia?

young age (<3 months), hypoxemia, vomiting and dehydration, underlying serious medical condition, complications (eg, empyema and abscess), or toxic appearance.

What is the most common cause of pediatric chronic kidney disease?

Congenital anomalies of the kidney and urinary tract are the most common cause of pediatric chronic kidney disease.

CHARGE syndrome

*C*oloboma *H*eart defects *A*tresia of the choanae *R*enal anomalies *G*rowth impairment *E*ar abnormalities/deafness CHD7

CHARGE syndrome

- choloboma - heart defects - choAnal atresia - retardation of growth/developmetn - GU abnomalities - ear abnormalities or deafness

Shigella flexneri - causes bloody diarrhea and what other discharge?

Pus/blood tinged vaginal discharge Use CTX to treat or axithromycin

Prophylaxis for the following patient with ARF: -Rheumatic fever + acute valvular issues (no residual).

-10 years -until 21 years of age.

Prophylaxis for the following patient with ARF: -Rheumatic fever + acute valvular issues (yes residual).

-10 years -until 40 years of age.

Prophylaxis for the following patient with ARF: -Rheumatic fever + no cardiac manifestations.

-5 years -until 21 years of age.

Klippel-Feil syndrome

-Chiari type 1 -congenital C-spine fusion - congenital heart disease, vertebral fusions at other levels, and hearing loss Before undergoing anesthesia or participating in contact sports, children with Klippel-Feil syndrome should undergo flexion and extension lateral cervical radiography to look for evidence of instability.

What is the formula for free water deficit calculation

0.6 × weight (kg) × ([serum sodium /140] -1)

2 year-old boy is brought to your office by his parents. He becomes very frustrated when he does not get his way. He typically makes poor eye contact. MOST appropriate intervention:

-a modified checklist for autism in toddlers (M-CHAT). -Autism can typically be diagnosed before age 3 years.

Patient is bipolar and taking lithium. She wants to have a baby. What are the effects of lithium during pregnancy?

-lithium levels greater than 1 mEq/L = congenital goiter -nephrogenic diabetes insipidus -transient hypothermia, cyanosis, bradycardia, shallow respirations, poor suck, hypotonia, and altered T waves on electrocardiography after birth

Ultrasound guided thyroid nodule biopsy woul d be indicated at which size of the thyroid nodules?

1 cm in diameter or risk factors for malignancy on history, physical examination, or ultrasonography Thyroid cancer occurs in about 25% of pediatric thyroid nodules, vs about 5% in adults.

What is the normal growth of head for the first six months?

1 cm/month

When do you triple your weight?

1 year old

What is the initial management of a child who presents with sudden onset of drooling, dysphagia, dysphonia, and dyspnea with high-fever?

Direct visualization of the airway under anesthesia. Diagnosis: Epiglottitis

Kocher Criteria

1) Assess Risk of Septic Arthritis in Children: A) Fever > 101.3 B) Non-Weight Bearing C) ESR > 40 D) WBC > 12,000 0: 0.2% Risk 1: 3% Risk 2: 40% Risk 3: 93% Risk 4: 100% Risk

Eagle Barrett syndrome ("prune belly")?

1) absent abdominal musculature 2) GU tract abnormalities 3) undescended testicles Oligohydramnios

What are good prognostic indicators for infantile spasms?

1. Idiopathic or cryptogenic 2. Age greater than 4 months 3. Treatment straight from onset 4. Initial response to treatment

Diagnostic criteria for AN, as defined by the Diagnostic and Statistical Manual of Mental Disorders (DSM)-V include:

1. Restriction of energy intake resulting in significantly low body weight 2. Intense fear of gaining weight 3. Severe body image distortion with denial of the seriousness of the illness Of note: Amenorrhea is no longer a criterion for the diagnosis of AN.

What is the normal between 6 months and 12 months?

1/2 cm/month

Greenstick fractures with angulation of greater than how many degrees require closed reduction?

15 degrees.

How much should water heater be set at?

120 F

Who do you immunize during transplant:

2 weeks before you can give inactivated vaccines 1 month before you should give live vaccines

Hib:

2, 4, 6 months Booster at 12 - 15 months

Average height velocity for the first year?

20 cm / year

Rota virus vaccine

3 dose series 2, 4, 6 months of age You can give a RV1 at 2 and 4 months You should not give rota > 15 wks (thats why last one they say 14 wk and 6 days); if you start 8 months you need to be done; if you start just finish it Dont give to SCID

Therapeutic hypothermia

33.5°C for 72 hours, to reduce the risk of morbidity and mortality.

Average head circumference at birth?

35 cm

Polio

4 doses are needed 2, 4, 6 -- through 18 months -- you can give them at 4 week intervals but stick to two month

When should you be in a booster seat until?

4 foot 9 inches or until adult seatbelt fits

When do you double your birthweight?

4 to 5 months

When do you double your birth height?

4 years old

If a parent has had a child with NTD, it is recommended that the mother receive a supplement with:

4,000 µg (4 mg)/d of folic acid beginning 1 month prior to planning to become pregnant and continuing through the first 3 months of pregnancy.

What is the most important way to prevent drowning?

4-sided fence around the pool, with a self locking and self latching gate

Average birth height is

50 cm

What red flag signs are far enough outside the norm for language:

6 months of age: lack of turning to sound or voice 9 months of age: lack of babbling consonant sounds 18 months of age: does not say "mama," "dada," or other names 24 months of age: failure to use single words 30 months of age: failure to use 2-word phrases 36 months of age: failure to use 3-word phrases

How many cafe-au-lait spots are needed for the diagnosis of NF-1

6.

When do secondary sexual characteristics and increased rate of linear growth occur in males and females

9 and 14 years in boys 8 and 13 years in girls

When should you be in a forward facing car seat?

> 2 yo AND weight more than 20 lbs

Alagille syndrome

Autosomal Dominant: Bile duct paucity with cholestasis, pulmonary artery stenosis, butterfly vertebrae, triangular shaped facies Hypertelorism and a murmur of peripheral pulmonic stenosis, JAG1 gene Affected children may also have posterior embryotoxon, an opaque ring around the cornea, due to thickening of the ring of Schwalbe

How long for the treatment of periorbital cellulitis?

A 10-day course of antibiotic therapy generally results in a successful outcome. In severe cases, a short duration of parenteral antibiotic therapy may be necessary.

What are the disadvantages of case series?

A case series, case-controlled series, or cohort study may show an association between patients and response to a new medication, but cannot be used to state causality.

Granuloma Annulare

A chronic or recurrent eruption of waxy, erythematous papules that tend to coalesce to form a ring (annular distribution). Management is to do nothing

Stages of hypertension

A normal blood pressure has systolic and diastolic blood pressures less than 120/80 mm Hg or the 90th percentile for the child's age, sex, and height, whichever is lower. Elevated blood pressure is defined as systolic or diastolic blood pressure between the 90th and 95th percentiles, or between 120/≤80 mm Hg and 129≤80 mm Hg. Stage 1 hypertension is defined as systolic or diastolic blood pressure between the ≥95th percentile to <95th percentile + 12 mm Hg, or 130/80 to 139/89 mm Hg (whichever is lower). Stage 2 hypertension is defined as systolic or diastolic blood pressure ≥95th percentile + 12 mm Hg or > 140/90 mm Hg (whichever is lower).

Smith-Lemli-Opitz Syndrome

AR inheritance 7-dehydrocholesterol reductase deficiency (inability to produce cholesterol) d/t mutation on DHCR7 gene 1 in 30,000 Diagnosis: measure serum 7-DHC (high, as no 7-DHC reductase to break down to cholesterol) Feats: FACE: microcephaly, broad nasal tip with anteverted nostrils, micrognathia, ptosis, epicanthal folds, cataracts, broad maxillary alveolar ridges, low-set ears, cleft palate BODY: short stature, hypotonia, syndactyly of 2nd/3rd toes, polydactyly, hypospadias HEART: VSD + PDA

A child with lower extremity weakness suspecting of DMD. What is the likely lab value that is highly specific?

A serum creatine kinase level of greater than 10,000 U/L is highly specific.

Treatment course for Kawasaki disease

A single dose of IVIG (2 g/kg over 10-12 hours) is most efficacious when given within 10 days of the onset of fever. A second dose is recommended if fever persists for 36 hours after the first dose. High-dose aspirin (80 mg/kg per day orally in 4 divided doses) is recommended while the patient is febrile; once the child's fever has abated, the dose is lowered to 3 to 5 mg/kg per day for 6 to 8 weeks.

What is considered an abnormal urine protein to Cr ratio?

A urine protein-creatinine ratio greater than 0.2 in a first-morning sample is abnormal. Studies have shown that the urine protein-creatinine ratio in a first-morning urine sample is as sensitive as 24-hour urine collection for detecting pathologic proteinuria.

What is the highest level of study design?

A well-designed, randomized control study of the medication referred to in the vignette might have demonstrated a cause-and-effect response to the new medication.

Erythema + drainage over lacrimal sac. What is the likely diagnosis?

Acute dacryocysitis: This is a medical emergency, requiring systemic antibiotics to cover methicillin-resistant Staphylococcus aureus, if prevalent in the community, and involvement of an ophthalmologist.

Which of the following is the most dangerous? (ANC = absolute neutrophil count)

Answer: A. ANC = 100 that is due to Kostmann syndrome. In Kostmann syndrome, children have myeloid arrest. Their myeloid stem cells are insensitive to G-CSF and fail to mature. They have no reserves, similar to a post-bone marrow transplant recipient, and are therefore extremely vulnerable to their own bacterial flora.

Pt treated with cephalosporin. Has right shoulder pain & elevated LFTs. What is the NBS?

Abdominal ultrasound.

What is the pathogenesis of Prune Belly Syndrome?

Abnormal intermediate and lateral plate mesoderm affecting abdominal wall development, mesonephric ducts, and genitourinary organs has been suggested as the underlying defect in prune-belly syndrome.

What is the treatment for acute confusional migraine?

Acetaminophen, ibuprofen, fluids, and caffeine can help speed recovery. Recurrent episodes treat with cyproheptidine.

Salter Harris type 1 fracture indicates bony injury at what structure?

Across the physis only.

When can you use acyclovir?

Acyclovir can also be used for postexposure prophylaxis, starting at 7 days after exposure, when passive immunoprophylaxis is not available.

What are the recommendation of GBS prophylaxis?

Adequate intrapartum antibiotic prophylaxis is defined as 5 million units of intravenous penicillin or 2 g of intravenous ampicillin or cefazolin administered at least 4 hours before delivery, then 2.5 to 3.0 million units of penicillin G or 1 g ampicillin or cefazolin every 4 hours until delivery.

When should children implement a weight loss plan?

After 9th grade.

When do you want to test levels of G6PD?

After time of hemolysis, because you will get false negative results if you test in hemolysis due to selective

Achalasia association

Alacrima (reduced ability to secrete tears) ACTH hormone resistance (Adrenal insuff) Achalasia Allgrove syndrome

What is the pathophysiology of aclochol induced hematemesis?

Alcohol-induced gastritis is caused by a direct toxic effect of alcohol that leads to gastric inflammation and eventual ulceration. Alcohol also decreases gastric motility, increasing contact time and thus worsening the injury.

What is the goal of pediatric hearing programs?

All states have universal newborn hearing screening programs, which aim to identify children with permanent congenital hearing loss by 3 months of age and begin interventions by 6 months of age.

Spirometry pattern of PVFD?

Although flattening of the inspiratory portion of the spirometric flow volume loop has been suggested as a diagnostic sign of PVFD, this finding has not been shown to be consistent. PVFD may coexist with and complicate asthma.

Definition of delayed puberty

Although the exact age of the onset of puberty varies, development of secondary sexual characteristics typically begins between ages 8 and 13 years in girls and 9 and 14 years in boys. Puberty is considered delayed in girls with no breast development by age 13 years, failure to menstruate by age 16 years, or more than 5 years between the initiation of breast development and menarche.

Recognize the anatomic effects of amniotic bands

Amniotic band syndrome results from interruption of the normal sequence of development during the third trimester. Amniotic band syndrome typically affects the arms or legs. Infants with amniotic band syndrome should be referred to a plastic surgeon for reconstructive surgery.

1 y/o has a cat bite. Swelling and tenderness 48 hrs later. What is the antimicrobial agent?

Amoxicillin-Clavulanate. Cover Pasteurella. Presentation is cellulitis within 24 hrs. Pen allergic? Use Azithromycin.

How do you treat orbital cellulitis?

Ampicillin-sulbactam and vancomycin to treat methicillin-resistant S aureus and other microorganisms associated with sinusitis. A 5- to 7-day course of parenteral antibiotic therapy (until the eye examination results are greatly improved), followed by 3 weeks of oral therapy is a reasonable approach.

Clostridium septicum

Associated with Cyclical Neutropenia

Pt with apparent leg length discrepancy, abnormal gait, and decreased hip range of motion. No constitutional symptoms. What is the next best step in management?

Apparent leg length discrepancy, abnormal gait, and decreased hip range of motion in a young child, in the absence of constitutional symptoms, suggests a dislocated hip -> hip radiographs

How do human milk and cow milk differ?

As compared to cow milk, human milk contains more available iron and carbohydrate, less protein and calcium, and similar amounts of fat.

What is the tx of acute bacterial sinusitis?

First-line antibiotics are amoxicillin or amoxicillin-clavulanate for a minimum of 10 days or for 7 days after the patient becomes symptom-free.

What is the definition of a BRUE?

BRUE refers to an episode in which an infant younger than 1 year of age exhibits 1 or more of the following: (1) cyanosis or pallor; (2) apnea, hypopnea, or irregular breathing; (3) hypertonia or hypotonia; and (4) altered mental status.

What is the pathophysiology of amniotic band syndrome?

Based on current theory, ABS is caused by a sequence of events leading to an interruption of normal development of the limb tissue. In animal models, the insult leading to ABS may be a vascular insult, hypoxia, or uterine trauma.

What is the diagnostic criteria for acute bacterial sinusitis?

Based on the 2013 guideline, criteria for acute bacterial sinusitis can be met with 1 of the 3 following patterns: persistent symptoms, severe symptoms, or worsening symptoms

16 yo adolescent + ankle injury. She has inversion injury and cannot bear weight. She has tenderness to palpation of anterior edge of lateral malleolus. No ankle instability. What is the NBS in management?

Begin range-of-motion exercises and wear a stirrup ankle brace Pearl: Ankle sprains should be treated with early mobilization, if tolerated Ankle support braces worn during sports can prevent ankle sprains

What do you consider in a In a neonate with meningitis who has been receiving appropriate antibiotic therapy for 3 days or more, persistent fever, or new focal neurologic findings including seizure?

Brain abscess

A baby born to a diabetic mom is normal with no symptoms of hypoglycemia. What is the next best step?

Breastfeed within an hour! An infant of a diabetic mother who has no symptoms of hypoglycemia should be fed, preferably by breast, within the first hour after birth and undergo a screening glucose measurement 30 minutes after the feeding.

Loxosceles reclusa

Brown recluse spider cytotoxic venom containing a substance similar to hyaluronidase. Children with brown recluse bites can display a spectrum of manifestations at the bite site, from minor local reactions (pain, erythema, and a blister at the bite site) to development of extensive tissue necrosis and ulceration. Affected children may also develop influenza-like symptoms (fever, chills, malaise, arthralgias, nausea, vomiting), intravascular hemolysis, disseminated rash, hematuria, and renal failure. No anti venom - management is more surgical debridement

What is the next best step in management in a patient who has congenital scoliosis?

Congenital scoliosis results from the presence of 1 or more congenital vertebral malformations (CVMs). Renal abnormalities are present in about one-third of children with CVMs. Therefore, children with CVMs should be evaluated with renal ultrasonography. If a child with CVMs exhibits signs of central nervous system dysfunction or midline cutaneous lesions overlying the spine, spinal imaging should be performed.

Children with vertebral anomalies should also undergo: Children with vertebral anomalies should undergo renal ultrasonography to look for underlying renal anomalies.

Children with vertebral anomalies should undergo renal ultrasonography to look for underlying renal anomalies.

SSRI in children

Consensus guidelines recommend fluoxetine as the preferred SSRI for the treatment of adolescent depression. Fluoxetine is also FDA-approved for treating childhood depression.

What is the most likely organism?

Chlamydia trachomatis The frequency of C trachomatis urethritis is almost 3 times that of N gonorrhoeae.

Patients with Hodkin lymphoma who got treated with anthracycline, bleomycin and neck radiation. What are the likely side effects?

CM -> doxorubicin Lung fibrosis -> bleomycin Neck radiation -> thyroid cancer

First step in management for thromboembolic cardioembolic stroke?

CT head - If magnetic resonance imaging is readily available, this could be considered instead of CT.

PNEUMOCCOCAL VACCINE (PCV13) ages

Can be given younger than 5 years 2, 4, 6 mo - and again 12 months

Who are the candidates for immunoprophylaxis?

Candidates for immunoprophylaxis include immunocompromised patients, certain neonates, and pregnant women. Immunocompromised patients include individuals with a congenital or acquired T-lymphocyte immunodeficiency, neoplasms affecting the bone marrow or lymphatic system, those who have received a hematopoietic stem cell transplant, and those receiving immunosuppressive therapy including prednisone at a dose of 2 mg/kg per day or more for 14 days.

A patient here in March with bronchiolitis sx + ear infection, what is the likely organism?

Human meta-pneumovirus - second most common cause of bronchiolitis - peaks after RSV season - ear infection commonly related

Treatment for PID

Ceftriaxone 250 mg intramuscularly (IM) × 1 dose (or cefoxitin 2 g IM and probenecid 1 g orally × 1 dose administered concurrently) PLUS Doxycycline 100 mg orally twice daily for 14 days WITH OR WITHOUT Metronidazole 500 mg orally twice daily for 14 days (for anaerobic coverage)

Know the most common clinical manifestation of a Pasteurella multocida infection

Cellulitis at the site of an animal bite that develops within 24 hours of injury)

Bacteria associated with brain abscess

Certain bacteria such as Citrobacter species including C koseri, Serratia marcescens, Proteus mirabilis, and Cronobacter sakazakii are particularly associated with brain abscesses.

What is Chiari 1 associated with?

Chiari I malformation can also be associated with spinal cord syrinx.

You are evaluating a 4-year-old boy who complains of "bouncing eyes." On downward gaze, there is downward nystagmus. What is the most likely diagnosis?

Chiari Malformation 1

How many polyps are OK to have as a juvenile prior to genetic testing?

Children with 5 or more juvenile polyps or any number of adenomatous intestinal polyps should be referred for genetic testing.

How does bicarbonate help with assessing degree of dehydration?

Children with bicarbonate levels higher than 15 mEq/L (15 mmol/L) are unlikely to be more than 10% dehydrated, and those with bicarbonate levels less than 14 mEq/L (14 mmol/L) are less likely to tolerate an oral fluid challenge.

Evaluation of a patient with hemi-hypertrophy

Children with hemihyperplasia, including Beckwith-Wiedemann syndrome, should undergo screening for embryonal tumors via serum alpha-fetoprotein measurement every 3 months until age 4 years, and abdominal ultrasonography every 3 months through age 7 to 8 years.

What is the likely diagnosis?

Cholelithiasis. Ceftriaxone has been associated with the development of gallbladder sludge or stones after prolonged use.

What can chronic hemiparesis in a former premature infant may be due to:

Chronic hemiparesis in a former premature infant may be due to periventricular leukomalacia or neonatal stroke.

Definition of chronic urticaria?

Chronic urticaria is defined by the presence of hives on most days of the week for a period of 6 weeks or longer. No need for allergy testing.

Pt presents with this rash that is worse with heat. What is the likely diagnosis?

Chronic urticaria.

Patient with recurrent pharyngitis by GAS. She keeps getting 3 to 4 infections in last six months. What are going to be effective treatment options?

Clindamycin - typically reserved for anaphylaxis. Amoxicillin may be resistant in this case. TMP-SMX has no streptococcus coverage. Augmentin, and cephalosporin.

What are other antibiotics associated with C-diff?

Clindamycin, amoxicillin, ampicillin, and the cephalosporins

Neonatal IVH

Clinical factors associated with increased risk of IVH include lower gestational age, pneumothorax, male sex, and bolus administration of normal saline or sodium bicarbonate. Protective factors include prenatal administration of betamethasone and delaying cord clamping until 30 to 60 seconds after birth.

Down's Syndrome Physical Exam features

Common dysmorphology in newborns with trisomy 21 includes small head with brachycephaly, epicanthal folds, upslanting palpebral fissures, small posteriorly rotated low-set ears, flat midface, Brushfield spots, small mouth, short thick neck, single transverse palmar crease, sandal toe, brachydactyly, and fifth finger clinodactyly.

DNAH11

Common gene affected in Kartagner's Syndrome

What are the facial features of a patient with Turner's Syndrome?

Common physical features include ptosis, epicanthal folds, low-set prominent ears, neck webbing, low posterior hairline, shortened fourth metacarpals, broad chest with widely spaced nipples, and multiple nevi.

10 yo boy with AMS, and confusion. Has negative tox/labs. Staring spells, and AO x 2. He has headache and vomiting on assessment. What is the most likely diagnosis?

Confusional migraine. Confusional migrane is characterized by an abrupt onset of altered consciousness.

Bone age = height age in what disorder.

Constitutional Growth Delay

What is the AAP recommendation on lithium and breast-feeding.

Contraindicated: -breastfed infants with lithium levels approaching 0.6 mEq/L have been noted to also experience cyanosis, lethargy, hypotonia, and poor feeding

What are Drugs that cause hypertension?

Corticosteroids, decongestants, nonsteroidal anti-inflammatory medications, herbal supplements, β-adrenergic agonists, erythropoietin, cyclosporine, tacrolimus, and stimulants (attention-deficit disorder medications).

Moebius syndrome

Cranial nerve V and XII sometimes VII) problems Tongue bilabial paresis, weakness Mask-like face, weakness or paralysis Many small chin, possible cleft Some: cond HL, lang delay due to hospitalization, articulation disorders

Labs show: A 2-year-old child presents for evaluation of gingival bleeding. Her mother reports that her gums bleed almost every time her teeth are brushed. Her review of systems is pertinent for a history of frequent nosebleeds. Pertinent laboratory findings are shown below. Poor aggregation in response to ADP, epinephrine, and collagen; Aggregation to ristocetin observed Diagnosis?

Glanzmann thrombasthenia. This disorder is due to an abnormality in the gene encoding the αIIb-β3 integrin fibrinogen receptor.

When would ophthalmology be indicated for dacrostenosis?

Dacrostenosis that persists after 6 months of age can be treated by an ophthalmologist with in-office lacrimal duct probing. Since most cases resolve spontaneously, some ophthalmologists prefer to wait until after 1 year of age to perform lacrimal duct probing under general anesthesia

Child has skin infection with MRSA and given clindamycin. What is the most common adverse reaction?

Diarrhea. C-diff can happen between days 4 and 9 of therapy, but may occur up to 10 weeks after completion of a course of antibiotics.

CVS

Diagnosis is based on the Rome IV criteria: Two or more periods of intense, unremitting nausea and paroxysmal vomiting that last for hours to days occur in a 6-month period. Episodes are stereotypical for each patient. Episodes are separated by weeks to months with return to baseline health between episodes. After appropriate medical evaluation, the symptoms cannot be attributed to another condition.

What may be a part of a post-ictal state in a patient with GTC?

Diffuse, symmetric hyperreflexia, sometimes with upgoing toes on plantar stroking, can be seen after a generalized tonic-clonic seizure, and does not necessarily imply a structural brain abnormality.

What is the pathophysiology of digital clubbing?

Digital clubbing is associated with hypoxemia and appears to increase in correlation with hypoxemia severity and chronicity.The pathogenesis of digital clubbing remains unclear. Proposed mechanisms include dilation of peripheral vessels, local deposition of platelet clusters, or stimulation of connective tissue growth.

Know the mode of transmission of Pasteurella multocida

Dog bites Cat scratches Cat Bites

Neonate born to a mother with no pre-natal care. Has a mean BP: 29. Baby is 35 weeks gestation. BP is 41/23. What is the next best step?

Dopamine infusion. You want your MAP = gestational age.

Swimming anticipatory guidance

Drowning is a common cause of death in children. Appropriately fitted personal flotation devices can significantly reduce the risk of drowning in open water and boating-related incidents. Swimming lessons have not been shown to decrease the risk of drowning in children younger than 4 years of age.

Varicella has a high-likelihood to be co-infected and get nec fasc from which organism?

Group A Strept

Common causes of SVC syndrome in children?

In childhood, SVC syndrome is usually caused by anterior mediastinal masses, enlarged mediastinal lymph nodes, and occlusion of the SVC itself.

What are the important steps for anti-bullying?

Effective antibullying programs incorporate improved student supervision, school-wide rules and antibullying policies, parent training, and collaboration between schools and families.

What is the treatment?

Empiric treatment is with azithromycin, 1 g orally, as a single dose for C trachomatis and ceftriaxone, 250 mg intramuscularly, for N gonorrhoeae.

What is the presentation for streptococcus infection?

Enlarged tonsils, tender cervical lymphadenopathy, palatal petechiae, and a strawberry tongue.

What is the criteria for enuresis?

Enuresis is diagnosed in children aged 5 years or older who void in bed or on clothes twice or more per week for 3 consecutive months.

Enuresis definition

Enuresis is diagnosed in children aged 5 years or older who void in bed or on clothes twice or more per week for 3 consecutive months. Primary enuresis occurs in children with no period of sustained dryness. Secondary enuresis is identified in children with a period of sustained dryness for 6 months (for nocturnal enuresis) or 3 months (for diurnal enuresis).

Pt who has vomiting of blood and splenomegaly. What is the likely cause of the bleeding?

Esophageal varices. In patients with upper GI bleeding, splenomegaly indicates the presence of portal hypertension and variceal bleeding.

Shwachman-Diamond syndrome

Exocrine pancreas dysfunction, including diabetes Short stature Variable hematologic abnormalities (pancytopenia, ^ HbF, myeloproliferative disorders) Radiologic skeletal changes (abnormal ribs, thorax, teeth, metaphyses and long bones) Other (psychomotor delay, renal abnormalities, icthyosis, Hirschsprung disease, hepatomegaly)

Who can you give VZig to:

Exposed people who are... Immunocompromised patients Pregnant women without immunity Hospitlized preemies of 28 or more weeks (w/o maternal immunity) Hospitilized preemies of < 28 weeks (regardless of maternal status) 5 days before and 2 days after Give varicella vaccine if exposed > 12 months

How does a Dacryocystocele present?

Firm, bluish mass below the medial canthus in the first few weeks of life and requires urgent referral to an ophthalmologist

What polyposis syndrome is associated with hypothyroidism?

FAP

What is the criteria for less than 48hr after delivery and healthy term newborns?

Gestation of more than 37 weeks meets criteria for discharge at or after 24 hours of age, with follow-up within 48 to 72 hours.

Who are low risk populations for UTI?

Girls older than 24 months without symptoms of UTI Uncircumcised boys older than 12 months Circumcised boys older than 6 months

What is the treatment for seborrheic dermatitis in young adults?

For affected areas of the skin, treatment is with a low-potency topical corticosteroid (eg, hydrocortisone 1% or 2.5%) or an agent active against yeast (eg, clotrimazole, miconazole nitrate, or ketoconazole) applied twice daily as needed. Scalp involvement is treated with an antiseborrheic shampoo containing pyrithione zinc, selenium sulfide, or ketoconazole

When do you evaluate a patient with a fever?

For children 3 to 6 months of age and those who have received fewer than 3 HiB and/or 3 PCV7 or PCV13 vaccines, the risk of occult bacteremia is higher. For these children, a complete blood cell count with differential should be obtained, and if the white blood cell (WBC) count is higher than 15,000/µL (15 × 109/L), a blood specimen should be sent for culture. It is also reasonable to perform chest radiography in children with no history of respiratory symptoms if the WBC is greater than 20,000/µL (20 × 109/L) and a reliable respiratory examination cannot be performed to assess for pneumonia.

Bacterial Sinusitis diagnostic criteria

For example, the diagnosis of acute bacterial sinusitis is clinical, based on 1 of the following 3 findings: persistent symptoms such as nasal discharge or daytime cough for at least 10 days without improvement; ill-appearing child with fever of at least 39°C and associated purulent nasal discharge and facial pain lasting for at least 3 consecutive days; and worsening symptoms or fever after initial improvement.

Hepatitis A pearls

For individuals who were exposed to hepatitis A virus within the last 2 weeks and are not eligible for hepatitis A virus vaccine, passive immunization with hepatitis A virus immune globulin is recommended. The hepatitis A virus vaccine is licensed for individuals 12 months to 40 years of age. The vaccine is used for postexposure prophylaxis for unvaccinated individuals 12 months to 40 years of age. Risk factors for hepatitis A virus infection in the United States include close contact with a child who attends a child care center, close contact with an international adoptee, contact with an infected individual, and international travel. Individuals with illicit drug use and men who have sex with men are additional groups at risk.

What is the relationship between athletic build and sports performance?

For most athletes, build has not been shown to predict success in sports and other physical activities.

HPV vaccine indications:

For those 9 to 14 years of age, a 2-dose schedule is recommended: the second dose is given 6 or 12 months after the first (if the series is initiated before the 15th birthday, 2 doses are administered) For those 15 to 26 years of age, a 3-dose schedule is recommended: the second dose is administered 1 to 2 months after the first; the third dose is given at least 6 months after the first.

What is the data on warm compresses?

Frequent warm compresses to the eye have been historically recommended, but evidence of effectiveness is limited.

____________________ presents with enlarged liver and spleen, osteopenia, focal lytic or sclerotic lesions of the bone, lung disease, and anemia with thrombocytopenia.

Gaucher disease Type 1 GD lacks central nervous system findings, while types 2 and 3 have progressive neurologic decline.

What is the most common fracture in children before 10?

Greenstick-type injuries are the most common fracture pattern in children, accounting for up to half of fractures in children before 12 years of age.

If a patient develops hypoglycemia without ketosis and permanent hepatomegaly after a 2- to 6-hour fast?

Growth hormone deficiency or hyperinsulinism should be considered.

Growth hormone indications

Growth hormone is approved by the US Food and Drug Administration (FDA) for children with idiopathic short stature whose height is more than 2.25 standard deviations (SDs) below the mean (<1.2%) and who are unlikely to catch up in height. A predicted adult height of less than 63 inches for boys or 59 inches for girls is considered a lack of expected catch-up growth. This boy does not meet these criteria, because his height is currently 2 SDs below the mean and his predicted adult height is well above 63 inches.

Tracheomalacia cough description.

Harsh and Brassy

In recent years what is the leading cause of noncrash automobile-related fatalities involving children?

Heat stroke/hyperthermia

Heriditary HyperTyrosinemia Type 1

Hereditary tyrosinemia type 1 is detected from newborn screening and typically presents in early infancy with profound liver failure. It can present in later infancy or early childhood with renal tubular dysfunction, rickets, and neurologic crises. Blood and urine succinylacetone levels are increased. An evaluation of serum amino acids will reveal elevated levels of methionine, tyrosine, and phenylalanine, and an evaluation of urine organic acids will reveal elevated levels of tyrosine metabolites. α-Fetoprotein levels will be significantly elevated. Hypoglycemia can present in association with profound liver failure.

How does systemic JIA present?

High, daily fever for at least 2 weeks, rash, lymphadenopathy, hepatosplenomegaly, serositis, and arthritis in 1 or more joints.

What are Immunosuppressive agents (corticosteroids and calcineurin inhibitors) associated with post-transplant?

Hypertension, metabolic syndrome, and diabetes mellitus. They also increase the risks of infection and posttransplant lymphoproliferative disorder.

Boy growth spurt age

IGS at 11.5 years of age, and reach a peak height velocity of 9.5 cm/year 2 years later

What is the dosage interval of IPV?

IPV at ages 2, 4, 6 to 18 months, and 4 to 6 years is currently recommended for all infants and children as part of a routine immunization series Every dose is 4 wks however minimal interval between dose 3 and 4 is 6 months

If OME persists longer than three months, what is the interval of time you should follow up?

If OME persists longer than 3 months, or if the clinician suspects hearing loss, language delay, or learning problems, watchful waiting is not appropriate and the child should undergo hearing testing. Children with OME lasting longer than 3 months who have normal hearing should have a routine follow-up every 3 to 6 months until the OME resolves, hearing loss is identified, or structural abnormalities of the middle ear are suspected.

What age do you wait to see majority of manifestations of NF-1?

If by the age of 10 years an individual has no eye findings (including Lisch nodules), axillary or inguinal freckles, neurofibromas, or other characteristic features, the diagnosis of NF1 is unlikely.

How do you proceed in management with low APGAR scores?

If the Apgar score is less than 7, scores should be assigned for 20 minutes or until the score is greater than 7. Apgar scores may be affected by prematurity, exposure to maternal medication, congenital neurologic conditions, and congenital anomalies.

Pneumococcal guidelines prior to splenectomy if you were not vaccinated:

If the patient has not previously been vaccinated with an age-appropriate regimen, the patient should receive the 13-valent pneumococcal conjugate vaccine (PCV13) followed by a dose of the 23-valent pneumococcal polysaccharide vaccine (PPSV23) after 8 weeks (and at least 2 weeks before splenectomy) and a second dose of PPSV23 after 5 years.

Who are the high risk kids for PCV13

If you are 6 to 18 years old and are: asplenic sickle cell HIV malignancy CSF leaks and cochlear implants You get a dose

Carseat laws

In 2011, the American Academy of Pediatrics issued updated recommendations to protect child passengers: Infants and young children should ride rear-facing in an appropriate CSS until at least 2 years of age or when they outgrow their CSS by weight or height. Once children are 2 years of age or they have outgrown their convertible car seat, they should ride forward facing in a CSS with a harness. Once children have outgrown their CSS with a harness by weight or height, they should use a belt-positioning booster seat until they have grown to properly fit into a seat belt. This typically does not occur until the child has reached a height of 4 ft, 9 in (145 cm) and is 8 to 12 years of age. Children who can use a seat belt should use one with both lap and shoulder restraints. All children younger than 13 years of age should ride in the back seat.

What is the diagnostic criteria of Sturge Weber?

In SWS, vascular malformations may involve the skin (PWS), brain (leptomeningeal angiomatosis that may cause seizures, stroke, and intellectual disability), and eyes (glaucoma). The diagnosis of SWS is clinical and requires involvement in at least 2 of these 3 areas.

Males and precocious puberty age:

In boys, puberty begins between the ages of 9 and 14 years. The first sign of puberty is testicular enlargement (>4 mL testicular volume, >2.5 cm in the long axis). Evidence of puberty in boys before the age of 9 years, or its absence by the age of 14 years, should prompt evaluation.

Ventilator strategies of SVC syndrome?

In fact, positive pressure ventilation compared to negative pressure spontaneous breathing may exacerbate derangements in ventilation, as forcing air past a fixed obstruction can be less effective than pulling air in with negative intrathoracic pressure.

Renal compensation in respiratory acidosis?

In the acute phase of respiratory acidosis, serum bicarbonate rises slightly (approximately 1 mmol/L for every 10 mm Hg of increased partial pressure of carbon dioxide [Pco2]) due to buffering by intracellular proteins. In the chronic phase of respiratory acidosis, over days or longer, serum bicarbonate rises approximately 3.5 mmol/L for every 10 mm Hg of increased Pco2, because of the increased reabsorption of bicarbonate at the renal tubular level.

What are the indications for paviluzimab

In the first year of age, healthy infants with a gestational age less than or equal to 28 weeks and 6 days, infants with hemodynamically significant congenital heart disease, and infants with chronic lung disease of prematurity can receive a maximum of 5 monthly doses of palivizumab.

Interpret EBV serologies

In the setting of an acute infection, serologic findings for both IgM and IgG against VCA are expected to be positive. The IgG against VCA persists for life. The IgM against VCA declines in the weeks to months that follow an acute infection and is replaced with IgG against EBNA. Acute or recent infection is suggested by the presence of IgM against VCA and the absence of IgG against EBNA. A past infection is suggested by the absence of EBV IgM and the presence of IgG against EBNA.

Ottawa ankle rules

Inability to bear weight Pain and point tenderness at tip and posterior area of malleolus

What is the definition of a greenstick fracture?

Incomplete fracture occurring at the diaphyseal-metaphyseal junction of her left radius, with the cortex remaining intact on one side.

Follicular ovarian cyst

Increased risk of functional ovarian cysts with smoking.

How does Heliox work?

Increases laminar flow/decreases turbulent flow. LOWER DENSITY than air (similar viscosity to air). The tendency for laminar vs. turbulent flow is described by the Reynolds number. Heliox's low density produces a lower Reynolds number and hence higher probability of laminar flow for any given airway. Laminar flow tends to generate less resistance than turbulent flow. Helium-oxygen mixture (heliox), less dense than 100% oxygen or a nitrogen-oxygen mixture, has a lower coefficient of friction for gas moving through the airways, and has been shown to improve ventilation and the delivery of albuterol to the small airways.

Indications for tympanostomy tubes

Indications for tympanostomy tube insertion include: Bilateral OME lasting at least 3 months together with conductive hearing loss Bilateral or unilateral OME lasting at least 3 months together with risk factors for speech, language, or learning problems (eg, neurodevelopmental disabilities, craniofacial anomalies) Bilateral or unilateral OME lasting at least 3 months together with symptoms or conditions such as pain, vestibular problems, tympanic membrane damage, or middle ear damage Recurrent AOM with OME at the time of assessment

Indications for varicella Ig

Individuals with primary or acquired immunodeficiencies Individuals who take immunosuppressive therapies Newborns if their mother developed varicella 5 days before to 2 days after delivery Premature infants born after at least 28 weeks of gestation who are exposed during hospitalization and have nonimmune mothers Premature infants born at less than 28 weeks of gestation or who weigh 1,000 g or less at birth who are exposed during hospitalization, regardless of the mother's immunity status Pregnant women Patients who receive varicella-zoster immune globulin should subsequently receive varicella vaccine, provided that no contraindications still exist, after at least 5 months have passed and they are 12 months of age or older.

When should you be in a rear facing car seat?

Infants < 2 yo AND weight less 20 pounds

What are the clinical findings of thanatophoric dysplasia?

Infants with thanatophoric dysplasia have dramatic shortening of long bones. However, their distal extremities have normal digits.

Your patient just had a BRUE what would place this patient at higher risk of having this again?

Infants younger than 2 months, those born at less than 32 weeks' gestational age with corrected gestational age less than 45 weeks, and those who had a previous event are considered to be at higher risk.

Juice guidelines

Intake of juice should be limited to, at most, 4 ounces daily for toddlers age 1-3. For children age 4-6, fruit juice should be restricted to 4 to 6 ounces daily; and for children ages 7-18, juice intake should be limited to 8 ounces or 1 cup of the recommended 2 to 2 ½ cups of fruit servings per day. No juice under 12 months old

What is the most common type of noncrash automobile injury sustained by children?

Involves the extremities (most commonly the fingers and hands) being crushed in closing automobile doors.

What is the most common nutritional def in the US?

Iron deficiency.

Pt after 4 weeks of iron therapy has correction of his anemia. What is the NBS?

Iron supplementation should continue for at least 2 months after the anemia has corrected to replenish the iron stores.

What is Sucrase-isomaltase deficiency?

It is a congenital deficiency of both sucrase and isomaltase enzymes, resulting in severe infantile diarrhea, failure to thrive, vomiting, and dehydration after starch/sugars are introduced into the diet. This deficiency is more common in native Greenlanders and Alaskans.

What is the management for a patient who is going to have a sinus fracture?

It is generally recommended that children with isolated sinus fractures be treated with a 1-week course of antibiotics. "Sinus precautions" (avoidance of nose blowing, swimming, straw use, and playing wind instruments) should be put in place. These patients should follow up with an otolaryngologist or plastic surgeon at 1 week; surgical intervention is rarely necessary.

K-Digo Criteria

KDIGO (Kidney Disease: Improving Global Outcomes) 2012 clinical practice guidelines diagnose CKD in children based on the presence of 1 of the following criteria: Glomerular filtration rate of less than 60 mL/min/1.73 m2 for greater than 3 months with implications for health, regardless of whether other CKD markers are present Glomerular filtration rate greater than 60 mL/min/1.73 m2 that is accompanied by evidence of structural damage or other markers of functional kidney abnormalities including proteinuria, albuminuria, renal tubular disorders, or pathologic abnormalities detected by histology or inferred by imaging

What is the triad of PCD?

Kartagener syndrome includes the triad of chronic sinusitis, bronchiectasis, and situs inversus totalis

What is the next best step if a patient has a midline dermal cyst?

Lesions involving the nose or midline scalp may be associated with intracranial extension, and lesions over the spine may be associated with spinal dysraphism. A dermal sinus tract may also occur. This appears as a pit, sometimes with a tuft of protruding hair or keratinous drainage. Sinuses involving the midline craniospinal axis, including the nasal bridge, occiput, and midline back, raise concern for connection with the underlying central nervous system and risk of meningitis. magnetic resonance imaging but occasionally computed tomography) to evaluate for possible intracranial extension

What are the different aspects of RMSF

Laboratory aberrations, including thrombocytopenia, hyponatremia, and elevated levels of hepatic transaminases, occur in over half of the cases of RMSF and result from endothelial injury and capillary leak.

Asian kid. 3 mo h/o bloating, gas, diarrhea. Went traveling 2 mo ago. Stool pH is 5.0. No weight loss, FOBT neg. What is the most likley test?

Lactose Breath Test.

A child with preceding upper respiratory infection, followed by fever, sore throat, and decreased oral intake. He devs neck stiffness and pain. What is the likely imaging modality to obtain?

Lateral Neck radiograph lateral neck radiography as abnormally thickened prevertebral soft tissue.

What are other syndromes that cause CALM

Legius syndrome: dominantly inherited condition characterized by multiple CALMs, axillary freckling, and macrocephaly

What phenotype may be very similar to CGD?

Leukocyte adhesion deficiency represents a group of genetic disorders that result in the failure to express proteins necessary for the normal trafficking of leukocytes to areas of infection. Without the ability to leave the bloodstream and enter an area of infection, the leukocytes are unable to contain infections. The resulting phenotype can be very similar to CGD.

What if an infant aged 30 d to 90 d presents with fever?

Likelihood of occult bacteremia in these patients is relatively low. Urine specimens should be sent for UA and Cx. For infants aged 30 to 60 days, obtaining a complete blood cell count with differential should be strongly considered. If WBC count is less than 5,000/µL (5 × 109/L) or greater than 15,000/µL (15 × 109/L), blood and cerebrospinal fluid cultures should be performed, and empiric intravenous or intramuscular antibiotics should be administered.

What eye finding is characteristic in NF-1?

Lisch Nodule

What is the next best step in management with someone with new diagnosis of NF?

Lisch nodules and optic glioma assessment by opthalmologist

What may exacerbate JME?

Lorazepam is used acutely to stop prolonged or repetitive seizures, but it is not an appropriate long-term anticonvulsant for JME. Phenytoin, oxcarbazepine, and carbamazepine can worsen seizures in JME, so these medications should be avoided.

LUMBAR syndrome

Lower extremity hemangioma, Urogenital anomalies and Ulceration, Myelopathy, Bony deformities, Anorectal malformations and Arterial abnormalities, and Renal anomalies (also described as SACRAL or PELVIS association)

What is the formula for MAP?

MAP = [ (2 x diastolic) + systolic ] divided by 3. MAP is considered to be the perfusion pressure seen by organs in the body.

Vaccines which interferes with TST

MMR (measles, mumps, rubella) vaccine may temporarily suppress tuberculin skin test (TST; a.k.a. purified protein derivative [PPD]) reactivity if it is given prior to the visit when the TST is performed because it is a live-attenuated virus vaccine. Therefore, if the measles vaccine is not administered during the same visit that a TST is applied, tuberculin testing should be delayed for 4-6 weeks after MMR vaccination in order for results to be considered valid. The same recommendation holds true for both varicella and live-attenuated influenza vaccines.

Ascending weakness of both legs with areflexia is suggestive of Guillain-Barré syndrome. What will lab studies show?

Magnetic resonance imaging of the lumber spinal cord sometimes shows nerve root enhancement and cerebral spinal fluid studies can show elevated protein with normal white blood cell count (cytoalbuminological dissociation).

What are manifestations of chromium deficiency?

Manifestations of chromium deficiency may include impaired glucose tolerance, although studies are conflicting.

What is the most commonly used illegal substance in the United States and worldwide?

Marijuana primary active component is delta-9-tetrahydrocannabinol (THC).

Side effects of Marijuana

Marijuana's other physiologic effects may include conjunctival injection, nystagmus, dry mouth, slurred speech, hunger, and ataxia.

Meniscus tear physical exam features

McMurray test, the external rotation loads (i.e., traps) the medial meniscus, whereas internal rotation loads (traps) the lateral meniscus. In the presence of a meniscal tear, pain along the joint line of the injured meniscus is elicited and is often associated with an audible or palpable "pop."

What are medications which are associated with Mg deficiency?

Medications associated with magnesium deficiency include proton pump inhibitors, diuretics, antibiotics (aminoglycosides, amphotericin, and pentamidine), calcineurin inhibitors, cisplatin, and antibodies targeting epidermal growth factor.

___________________________ occur when a child extremities is pulled or twisted forcibly or when the child is shaken.

Metaphyseal chip fractures, also known as corner or bucket-handle fractures, occur when a child's extremity is pulled or twisted forcibly or when the child is shaken.

What is the gold standard of assessing for middle-ear effusion?

Middle-ear effusion is best diagnosed with pneumatic otoscopy and tympanometry may be used to confirm the diagnosis.

Pearson Syndrome

Mitochondrial Disease Infantile pancreatic insufficiency, pancytopenia, lactic acidosis

Statistics of OME?

Most episodes of OME resolve without intervention, but nearly one-third of children will have recurrence and 5% to 10% of children have persistent effusion lasting at least 1 year.

A 20-month-old, previously healthy girl is preparing to go on a 5-week trip to Lagos, Nigeria, as part of an organized church group trip. Her mother is concerned about their risk of acquiring poliomyelitis in Nigeria. The child was born in the United States and received inactivated polio vaccination at 2, 4, and 15 months of age. Her mother was also born in the United States and received her complete polio vaccination series as a child. The girl's physical examination findings are normal. Who gets vaccinated?

Mother - she has documented immunity prior and then now needs a booster dose; if she did not have immunizations prior she would need 3 shot series

What are other organism that can cause NGU?

Mycoplasma genitalium, Ureaplasma urealyticum, Trichomonas vaginalis, herpes simplex virus (HSV), and adenovirus

What is the first step in management?

NG tube and hemodynamic stability.

What is the management of brown recluse spider bites?

NSAIDS and proper wound therapy It is common for these patients to have systemic signs of infection

What is the management of HSP?

NSAIDs

Sotos Syndrome

NSD1, 5q34, AD macrocephaly, tall, ID FISH, MLPA, NSD1 sequencing

A 5 wk otherwise healthy infant p/w excessive tearing, most frequently unilateral, but sometimes bilateral. What is the first line treatment?

Nasolacrimal duct massage several times per day. Massage involves a caretaker using a clean finger to place firm pressure over the lacrimal sac, stroking downward.

When do you use oral corticosteroids vs. topical steroids for allergic contact dermatitis?

Oral steroids are often used to treat rhus dermatitis when there are large reactions that lead to swelling around the eyes or genitals.

Treatment of uncomplicated posttraumatic periorbital cellulitis

Oral antimicrobial agents (eg, clindamycin or trimethoprim-sulfamethoxazole in combination with amoxicillin or linezolid) that treat S aureus (including methicillin-resistant S aureus) and group A Streptococcus.

What may be a fetal DNA screen prior to CVS or amniocentesis?

Noninvasive prenatal testing is a newer technology that uses a sample of the mother's blood to isolate fragments of the fetus's DNA to screen for trisomies and sex chromosome abnormalities, with a high detection rate. This screening test can be performed as early as 10 weeks of gestation. Diagnostic confirmation via amniocentesis or chorionic villus sampling is recommended if a screening test result is positive.

What is the Schamroth sign?

Normal diamond-shaped window seen when dorsal surfaces of the terminal phalanges of opposite index fingers are apposed is absent.

What criteria do we use if we are not going to get brain CT?

Normal neurologic examination findings Normal mental status Normal behavior as noted by caregiver No loss of consciousness No vomiting No severe headache No evidence of skull fracture (no nonfrontal scalp hematoma for children younger than 2 years of age) No signs of basilar skull fracture No high-risk mechanism No concern for inflicted injury

PPSV23

Not in healthy children Children at age 2 Make sure it is 8 weeks after PCV 13 You need 5 years later another PPSV (NOT cochlear kids)

What is the normal BP cuff size position?

Note 40% of Circumference at Mid-Point

What are the laboratory studies for Orthostatic Proteinuria?

OP occurs during the day when the patient is active and disappears when the patient is supine/asleep for at least 2 hours.

An adolescent girl who has closed head injury - very brief loss of consciousness - no focal neuro deficits. What is the next best step in management?

Observe in ED

What is the first line agent for JME?

Of the choices, valproate is the best medication to treat juvenile myoclonic epilepsy.

Infants with Down's syndrome will have likely what sort of CHD?

Of those infants, 40% to 50% will have an atrioventricular canal-type defect.

What are the stats of ophalitis?

Omphalitis is a rare complication affecting less than 1% of all neonates born in the United States. However, it is associated with rates of morbidity and mortality as high as 85%.

After CT what is the next best step in management?

Once embolic ischemic stroke is confirmed, there should be prompt investigation for a source. If there is any residual thrombosis in the heart or distal veins, anticoagulation therapy should be considered to prevent further emboli.

Diabetes Screening

Options for screening for diabetes include obtaining a: fasting plasma glucose level (≥125 mg/dL [6.9 mmol/L] is diagnostic) 2-hour plasma glucose level during a 75-g oral glucose tolerance test (≥200 mg/dL [11.1 mmol/L] is diagnostic) OR hemoglobin A1c level (≥6.5% is diagnostic)

Infants born to mothers with diabetes:

Other than hypoglycemia, associated complications of gestational diabetes include macrosomia, prematurity, asymmetric septal hypertrophy causing cardiac outflow obstruction, respiratory distress syndrome, caudal regression syndrome, and small left colon syndrome.

What does NBS for hearing include?

Otoacoustic emission (OAE) or auditory brainstem response testing.

ABP Content Specifications(s) Identify the major physiologic consequences associated with marijuana use/abuse Recognize the major behavioral consequences of marijuana use/abuse Manage a patient with poor school performance.

PREP Pearls 1. Illicit drug use should be considered in the differential diagnosis of an adolescent with new onset academic underachievement. 2. Marijuana is the most commonly used illicit drug and has negative immediate and long-term behavioral and health consequences. 3. Marijuana's physiologic effects include tachycardia, increased blood pressure, decreased pulmonary function, increased risk of lung cancer, conjunctival injection, nystagmus, dry mouth, slurred speech, hunger, and ataxia. 4. Neurobehavioral consequences of marijuana use include poor executive function, decreased concentration, memory impairment, distorted perception, drowsiness; impaired cognition, judgment, and coordination; and reaction time. 5. Mental health problems such as anxiety, depression, and schizophrenia may worsen with heavy use of marijuana.

ABP Content Specifications(s) Recognize the clinical presentation of chronic granulomatous disease Plan and interpret the results of laboratory evaluation in a patient with chronic granulomatous disease

PREP Pearls A defect of the innate immune system should be suspected in a child presenting with frequent bacterial infections. Chronic granulomatous disease (CGD) is the most common disorder of neutrophil function. The gold standard for diagnosing CGD is testing of the phagocyte (neutrophil) oxidative burst by flow cytometry.

ABP Content Specifications(s) Understand the usefulness of maternal blood screening in prenatal diagnosis

PREP Pearls A maternal quadruple screening result showing an elevated maternal serum α-fetoprotein (AFP) with normal human chorionic gonadotropin (hCG), inhibin A, and unconjugated estriol levels (μE3) is highly suggestive of an open neural tube defect. Maternal quadruple screening in pregnancy measures serum levels of AFP, unconjugated estriol, the β component of hCG, and inhibin A. Maternal quadruple screening is used in the second trimester of pregnancy (between the 15th and 20th week) to assess the risk of a fetal chromosomal abnormality such as trisomy 21, trisomy 18, or an open neural tube defect/anencephaly. If a prenatal screening test is positive for an increased risk of a chromosome abnormality, diagnostic confirmation via amniocentesis or chorionic villus sampling is recommended.

ABP Content Specifications(s) Understand the evaluation for developmental dysplasia of the hip. Recognize the clinical findings associated with developmental dysplasia/subluxation of the hip Plan the appropriate diagnostic evaluation of developmental dysplasia/subluxation of the hip in patients of various ages

PREP Pearls For an ambulatory child, a unilateral toe-walking gait and limited hip range of motion suggests a hip dislocation. Ultrasonography is the preferred imaging method to evaluate hip dysplasia in a child younger than 5 to 6 months. Radiographic evaluation is the preferred imaging method to evaluate hip dysplasia in a child older than 6 months

ABP Content Specifications(s) Plan appropriate umbilical cord care

PREP Pearls In the developed world, the umbilical cord should remain dry after birth without additional treatment. Application of isopropyl alcohol to the umbilical cord does not decrease the incidence of omphalitis

ABP Content Specifications(s) Differentiate between type 1 and type 2 diabetes

PREP Pearls Insulin therapy is indicated for the child or adolescent with newly diagnosed type 2 diabetes if the presenting glucose is greater than 250 mg/dL or the hemoglobin A1c is greater than 9%. Features making type 2 diabetes more likely than type 1 diabetes include non-white race, obesity, signs of insulin resistance (eg, acanthosis nigricans), insidious symptom onset, and lack of ketosis. Features making type 1 diabetes more likely include white race, lack of obesity, acute onset of symptoms, associated autoimmunity, ketosis, and presence of diabetes-associated antibodies.

ABP Content Specifications(s) Recognize the clinical findings associated with juvenile myoclonic epilepsy, and manage appropriately

PREP Pearls Juvenile myoclonic epilepsy is diagnosed based on the clinical features of myoclonic jerks of the upper extremities on awakening and generalized tonic-clonic seizures, and in some cases, a history of absence seizures. Juvenile myoclonic epilepsy has a characteristic electroencephalogram pattern of 4 to 6 Hz spike wave discharges. Valproate is a first-line anticonvulsant for juvenile myoclonic epilepsy; phenytoin, oxcarbazepine, and carbamazepine can exacerbate seizures in this syndrome and should be avoided.

ABP Content Specifications(s) Formulate an age-appropriate differential diagnosis in a patient with constipation Plan the appropriate management of a patient with constipation Understand the action of laxatives, stool softeners, and lubricants in a patient with constipation

PREP Pearls Lactulose is an osmotic agent used to treat constipation. Constipation has a broad differential diagnosis that differs between infants and children. Dietary management plays a significant role in the successful management of constipation.

ABP Content Specifications(s) Plan the early discharge of a newborn infant, including follow-up evaluation Plan the management of a neonate whose mother has abnormal prenatal laboratory findings

PREP Pearls Minimum criteria must be met for early discharge of a newborn (< 48 hours after delivery). Adequate intrapartum antibiotic treatment for the prevention of group B streptococcal disease is required to meet early discharge criteria. The hospital stay for a healthy term newborn and mother should be long enough to identify problems in either and to ensure that the mother is able to care for herself and her newborn at home.

Plan the appropriate clinical and diagnostic evaluation of laryngeal and vocal cord disorders

PREP Pearls Paradoxical vocal fold dysfunction (PVFD) is a common and often underrecognized etiology of exertional dyspnea. Paradoxical vocal fold dysfunction may be triggered, and or exacerbated, by comorbidities such as laryngopharyngeal reflux, allergic rhinitis, rhinosinusitis, or untreated obstructive sleep apnea. The diagnosis of PVFD may be confirmed on direct laryngoscopic visualization demonstrating adduction of the vocal cords with a classic posterior chink-like opening.

ABP Content Specifications(s) Understand the relationship between varicella and herpes zoster infection Plan appropriate control measures to prevent the spread of varicella and herpes zoster Understand the epidemiology of varicella-zoster virus Recognize the clinical features associated with varicella and herpes zoster infections in normal and immunocompromised children of various ages

PREP Pearls Passive immunoprophylaxis after exposure to varicella is indicated in immunocompromised patients, certain neonates, and pregnant women. Varicella-zoster immune globulin should be given as soon as possible after exposure or up to 10 days after exposure to varicella. For individuals that are nonimmune and exposed but otherwise do not meet criteria for immunoprophylaxis, varicella vaccine can be used if the individual is 12 months of age or older and the vaccine is not contraindicated.

ABP Content Specifications(s) Recognize the risk factors for the development of pseudomonal infections Recognize the clinical manifestations of pseudomonal infections and manage appropriately

PREP Pearls Piercings of the upper ear helix may cause a suppurative perichondritis, frequently caused by Pseudomonas aeruginosa, instead of Staphylococcus aureus. Systemic treatment with an antipseudomonal agent like ciprofloxacin is required. Surgical drainage, debridement, and reconstruction of the ear may be necessary.

ABP Content Specifications(s): Understand the effect of rapid body changes on an adolescent's sense of self Understand features associated with an adolescent's search for identity

PREP Pearls Psychosocial developmental during adolescence involves the achievement of a mature self-identity, mature sexuality and independence. Early pubertal changes in girls have been associated with lower self-esteem and poor body image.

ABP Content Specifications(s) Understand the validity hierarchy for study design and study type Understand the principles and application of study design Understand the uses and limitations of randomized clinical trials Understand the uses and limitations of controlled clinical trials

PREP Pearls Randomized controlled studies can provide evidence for causality. Observational studies, including case series and case control studies, can demonstrate association but not causality.

ABP Content Specifications(s) Recognize the clinical findings associated with peritonsillar abscess Recognize the clinical findings associated with retropharyngeal abscess

PREP Pearls Retropharyngeal abscesses present with the insidious onset of symptoms including fever, sore throat, neck stiffness, and in more severe cases, tachypnea, drooling, and stridor. Retropharyngeal abscesses are most common in children younger than 6 years of age. Lateral neck radiographs of retropharyngeal abscesses show thickened prevertebral soft tissues. Peritonsillar abscesses, most common in adolescents and young adults, present with fever, sore throat, muffled voice, and dysphagia. Ultrasonography or computed tomography is useful in making the diagnosis of peritonsillar abscesses

ABP Content Specifications(s) Differentiate the causes of acute, subacute, and chronic weakness Understand the benefits and limitations of neurodiagnostic tests in the evaluation of weakness

PREP Pearls Risk factors for stroke, such as unrepaired atrial septal defect, should raise clinical suspicion for stroke as a cause of new neurological abnormalities.

ABP Content Specifications(s) Recognize the clinical findings associated with superior vena cava syndrome

PREP Pearls Superior vena cava syndrome in previously healthy children is usually associated with a malignant anterior mediastinal mass. Extreme caution should be taken prior to sedating, anesthetizing, or initiating positive- pressure ventilation in a child with superior vena cava syndrome or anterior mediastinal mass.

ABP Content Specifications(s) Recognize disorders commonly associated with digital clubbing

PREP Pearls The finding of digital clubbing suggests an underlying medical condition. The classic clinical phenotype of primary ciliary dyskinesia (PCD) includes a history of neonatal respiratory distress in more than 80% of affected individuals, daily nasal congestion, and wet cough, starting in the early neonatal period, and chronic otitis media and sinus disease beginning in early childhood. Primary ciliary dyskinesia is often unrecognized; this may result in delayed diagnosis, delays in access to optimal therapies, and disease-associated morbidity and mortality A significant proportion of individuals with heterotaxy syndromes have symptoms suggestive of PCD. Kartagener syndrome includes the triad of chronic sinusitis, bronchiectasis, and situs inversus totalis.

ABP Content Specifications(s) Recognize the clinical findings associated with contact dermatitis

PREP Pearls Topical steroids are the mainstay of treatment for contact dermatitis; oral steroids are indicated only when there are large reactions around the eyes or the genitals. Fluid from ruptured vesicles in rhus dermatitis does not lead to additional areas of skin being affected.

ABP Content Specifications(s): Plan the appropriate diagnostic evaluation of closed-head injury and brief loss of consciousness Plan the appropriate physical and laboratory evaluation of head injury, including serial evaluations of the patient's status Recognize the immediate life-threatening complications of closed-head trauma

PREP Pearls While computed tomography (CT) of the brain is a rapid and accurate way to identify intracranial injuries in children after head trauma, use of this diagnostic modality has downsides, including exposure to ionizing radiation, identification of minor lesions or incidental findings with unclear clinical importance, the need for sedation for younger or uncooperative pediatric patients, and significant increases in healthcare costs The goal of pediatric providers should be to identify children with clinically important intracranial injury after head trauma to prevent deterioration and secondary brain injury, while limiting unneeded radiographic imaging in children at low risk. For patients falling into the "intermediate risk" category for intracranial injury, observing patients clinically before obtaining computed tomography of the brain allows providers to selectively image only those whose symptoms worsen or fail to improve.

10-year-old female soccer player on exam has inspiratory stridor and expiratory wheezing. The girl's coach has noted that the girl develops a "squeaky" voice during these episodes of exercise intolerance. What is the likely diagnosis?

Paradoxical vocal fold dysfunction

What is the pathophysiology of paradoxical vocal fold dysfunction?

Paradoxical vocal fold dysfunction is defined by adduction of the vocal cords during inspiration, or during inspiration and expiration, with preservation of a posterior region of glottic opening known as a "posterior glottic chink."

What is the classic presentation of PVFD

Patients experience episodes of dyspnea, wheezing, stridor, and/or throat tightness; these symptoms mimic those of asthma, but typically respond poorly to asthma therapies. Typically, PVFD is triggered by exertion

When do you vaccinate - pre or post vaccination?

Patients undergoing elective splenectomy should be vaccinated against encapsulated organisms prior to the splenectomy.

What is the treatment for a patient who presents with ARF?

Penicillin V 250 mg BID.

PHACES syndrome

Posterior fossa (dandy walker) Hemangiomas Arterial anomalies Coarctataion, cardiac defects Eye abnormalities Subglottic hemangiomas

What thresholds would make you go directly to insulin therapy?

Plasma glucose is greater than 250 mg/dL and hemoglobin A1c is greater than 9%, initial management with insulin is indicated.

gasoline ingestion what is the most likely complication?

Pneumonitis volatile means how well can they get into air 2-6 hrs after ingestion of gasoline: watch for tachypnea, wheezing & rales.

What is the empiric post exposure prophylaxis following sexual assault:

Postexposure prophylaxis following sexual assault (CDC recommendations): Empiric treatment for C trachomatis, N gonorrhoeae, and trichomoniasis infection Emergency contraception for women Hepatitis B immunization without hepatitis B immunoglobulin (if the hepatitis status of the assailant is unknown and the survivor has not previously been immunized) Human papillomavirus immunization (if the survivor has not previously been immunized) HIV postexposure prophylaxis depending on risk (see: CDC Sexually Transmitted Diseases Treatment Guidelines, https://www.cdc.gov/std/tg2015/default.htm and below)

NBS if you find an external ear anomaly?

Renal ultrasonography should be performed if an infant has external ear malformations plus at least 1 other significant clinical finding.

Plan the appropriate initial and follow-up management of otitis media with effusion in patients of various ages, including when complications occur. Recognize conditions (including allergic rhinitis, adenoidal hypertrophy, eustachian tube abnormalities) associated with otitis media with effusion.

Prep pearls: Otitis media with effusion (OME) may occur spontaneously, be the result of acute otitis media, or associated with other conditions. Conditions associated with OME include allergic rhinitis, adenoidal hypertrophy, eustachian tube abnormalities, and craniofacial anomalies. In children with OME who are not at risk for language delay, watchful waiting with follow-up in 3 months is appropriate. Children with OME persisting longer than 3 months; suspected hearing loss, language delay, or learning problems; and those at risk for language delay should undergo hearing testing.

Formulate a differential diagnosis of urinary tract obstruction Understand the various causes of urinary tract obstruction

Presence of bladder distention or hydronephrosis on ultrasonography is suggestive of urinary obstruction. Renal replacement therapy (eg, intermittent hemodialysis, continuous hemofiltration, and peritoneal dialysis) is considered for patients with renal failure and complications of volume overload, hyperkalemia, uremia, symptoms associated with uremia), severe acidosis, or an inability to provide adequate nutrition.

What is the specific treatment for iron def?

Presumptive iron deficiency anemia may be treated with oral (elemental) iron at a dose of 3 to 6 mg/kg per day for 4 weeks.

IQ vs. ID

Psychoeducational tests, such as IQ tests and achievement tests, generally have a mean score of 100 and standard deviation (SD) of 15. The average range is within 1 SD of the mean (85-115). IQ test scores greater than 2 SD below the mean (<70) in both cognitive and adaptive measures are in the intellectually disabled range. A learning disability may be defined as a meaningful discrepancy between intelligence scores and achievement scores, low achievement in the setting of at least low average intelligence, or a student's failure to respond to evidence-based educational interventions.

What is the criteria for diagnosis of HSP?

Purpura or petichiae PLUS arthritis or arthralgia, abdominal pain, histopathology demonstrating immunoglobulin A deposition, or renal involvement demonstrated by hematuria or proteinuria

Reynolds equation for turbulence?

Re = Vp2r/n V = velocity p = density r = radius n = vicosity

In which cells of the hematopoietic system are enzyme levels of G6PD the highest?

Reticulocytes.

What are fractures that involve the epiphysis or growth plate known as?

Salter Harris.

Goldenhar syndrome

Same as fetal warfarin syndrome Facial asymmetry and hypoplasia Microtia, ear tag Cleft lip/palate Hypoplastic vertebrae CVS Frequent (35%) VSD, TOF

A baby born to a diabetic mom has jitteriness. What is the next best step?

Screening glucose values should be obtained in infants with symptoms and intravenous glucose initiated if the value is less than 40 mg/dL (2.2 mmol/L).

What is the criteria for secondary enuresis?

Secondary enuresis is identified in children with a period of sustained dryness for 6 months (for nocturnal enuresis) or 3 months (for diurnal enuresis).

Loeys-Dietz syndrome

Similar to Marfans, but worse Verty tortuous vessels Hypertelorism Bifid uvula Bulla Skin laxity

What is the mechanism by which the perichondrium receives nutrition?

Since cartilage is avascular, it is dependent on the perichondrium for its nutrients and oxygen

How does skull growth occur in relation to suture lines?

Skull growth is perpendicular to suture lines.

What are clinical features that make you suspicious for Turner's Syndrome?

Small size at birth, history of recurrent otitis media, difficulty with math, short stature with declining growth velocity, epicanthal folds, high-arched palate, and multiple nevi.

How is Pasteurella different from GAS or Staph aureus?

Staph aureus and GAS has 1-2 day onset.

Autoimmune Hepatitis

Suspect autoimmune hepatitis (Type I) in female adolescents with sudden onset of jaundice, negative viral studies, no medication use, and no stones in the gallbladder or biliary tree. Type I presents between 10-20 years of age and is more common in females. Antinuclear antibodies (ANA) and anti-smooth muscle antibodies are usually positive. Type II autoimmune hepatitis is normally seen in younger children, and it is typically more severe in presentation. Antiliver kidney microsomal antibody is usually positive.

What is the presentation of PTA?

Symptoms include fever, sore throat, muffled or "hot potato" voice, and dysphagia, leading to decreased oral intake. Patients may experience pain referred to the ipsilateral ear and may also have trismus.

Crouzon Syndrome

Syndrome presenting with multiple sutures fuse early Craniosynostosis - early fusion of the cranial bones, small maxillary structure, hypertelorism, protrusion of eyeballs, strabismus, parrot like nose, malocclusion class III; caused by autosomal dominant inheritance

Apert Syndrome

Syndrome presenting with syndactyly (fused fingers), abnormal cranial bone growth, hearing loss, class III malocclusion, 25% of cases with cleft; caused by autosomal dominant mutations FGR2 at 10q25-26

What are the clinical manifestations of JME?

Sz + jerking outside of the seizure period

A 20 month old patient who is about to get a vaccine starts head banging. Normal development; what is the next best step?

Take them to a safe place, support the child. No need to have punishment - head banging can be normal in isolation.

What are the components of temperament?

The 9 attributes of temperament are: activity level, rhythmicity or regularity, initial approach/withdrawal, adaptability, intensity, mood, persistence/attention span, distractibility, and sensory threshold.

First step in management for child with intellectual delay?

The American Academy of Pediatrics and the American College of Medical Genetics recommend chromosomal microarray and fragile X testing as first-tier testing for children with intellectual disability.

What is the characteristic pattern on EEG for JME?

The EEG in untreated individuals is typically abnormal with a 4-6 Hz generalized polyspike and slow wave discharge. The exact mode of inheritance is unclear. Some patients with juvenile myoclonic epilepsy have a positive family history of epilepsy. This is a lifelong condition, and seizures are generally well controlled with levetiracetam or valproic acid.

High risk conditions requiring two lipid profiles

The average of these 2 results should be used to dictate next steps.

A 14-year-old boy presents with back pain that has gradually worsened over the past several months. On physical examination, he is noted to have prominent kyphosis, which is obviously greater than that of the normal thoracic spine. Mild-to-moderate pain is elicited over the apex of the kyphosis. When asked, the patient is unable to voluntarily correct the deformity. A radiograph is performed. Which of the following radiographic findings is most likely to be identified in this patient?

The boy has clinical signs and symptoms of structural kyphosis (Scheuermann disease). Characteristic findings on a plain lateral radiograph demonstrate anterior wedging of ≥ 5° in 3 or more consecutive vertebral bodies at the apex of the deformity in the thoracic or thoracolumbar region, in association with vertebral endplate abnormalities. Schmorl nodes, caused by anterior protrusion/herniation of the nucleus pulposus into the vertebral body, may also be identified on a plain radiograph. The normal range of thoracic spine curvature is between 20° and 40°. Individuals with Scheuermann disease have fixed kyphotic deformities of ≥ 45°.

What is the EEG findings of JME?

The diagnosis of JME is based on the presence of these clinical features, an electroencephalogram showing the characteristic pattern of a 4 to 6 Hz spike and slow wave discharges in an otherwise typically developing adolescent. In typical cases, brain imaging is not indicated.

Glaucoma triad

The classic presentation of congenital glaucoma includes excessive tearing of the eye, photophobia, and frequent blinking of the eyelid due to muscle spasms. Less than 30% of patients will present with this classic triad.

What combination therapy can detect open neural tube defect?

The combination of maternal quadruple screening along with a targeted second-trimester ultrasonography can detect an open neural tube defect in 95% of cases.

A patient is 6 weeks old and has drop in hemoglobin. Normal appearing child. What is the mechanism?

The concentration of fetal EPO is highest in third trimester and rapidly drops with the sudden increase in blood oxygen at birth. Low EPO result in a slow decrease in the hemoglobin in the first 8 to 12 weeks.

Maternal Quadruple Screening Findings in Fetal Anomalies/Trisomies.

The detection rate for trisomy 21, neural tube defects, and trisomy 18 is approximately 80% with maternal quadruple screen alone, with a 5% false-positive rate.

What is the dose of folate during pregnancy?

The discovery of folic acid deficiency contributing to NTD risk led to a public health initiative to supplement all women preconceptually for 1 month and postconceptually for at least 3 months with 400 to 800 µg/d of folic acid, which led to a greater than 75% reduction in the incidence of NTDs.

What are the clinical features of congenital talipes equinovarus?

The features of talipes equinovarus include a high arch, adducted position of the forefoot, hindfoot varus, and plantar flexion contracture.

Gun Violence summary

The following statements summarize important data that can be used in counseling or intervention strategies: Firearm-related injuries are often fatal; primary prevention is essential. Safer storage of guns and ammunition reduces injuries. Access to a gun increases the likelihood that intentional injuries to oneself or others is lethal. Access to a gun and unsafe storage practices create risk of serious unintentional injury and death. There are ethnic and racial disparities in the frequency of gun violence and injury. White children experience the least gun violence and injury, and black children experience the most. Youth 15 to 19 years of age incur firearm-related injuries about 2.5 times more frequently than the general population. Children 0 to 4 years of age have the highest proportion of unintentional firearm-related deaths. The incidence of unintentional injuries from guns increases with age. Guns are the leading mechanism of intentional deaths for children aged 5 years and older. Access to guns increases the mortality rate from suicide attempts to 90%. For children and adolescents, most firearm-related injuries and deaths involve a handgun; however, in rural areas, a large number of unintentional injuries and suicides involve long guns.

What is the gold standard testing for CGD?

The gold standard for diagnosing CGD is testing the phagocyte oxidative burst with flow cytometry.

Initial nephrotic syndrome tx

The initial episode of NS is treated with oral steroids (60 mg/m2/d for 4-6 weeks, followed by 40 mg/m2/d given every other day for 2-5 months, with gradual tapering). The most important determinant of renal prognosis in idiopathic NS is steroid responsiveness.

What is the treatment of PVFD?

The mainstay of treatment for PVFD is laryngeal control therapy. Laryngeal control therapy is also referred to as respiratory retraining therapy and is performed by a licensed speech and language therapist. This treatment, demonstrated to be effective in 95% of patients, focuses on diaphragmatic breathing and laryngeal relaxation techniques.

Stat on pediatric closed head injuries.

The majority of pediatric closed head injuries are minor, though some can be life-threatening; more than 3,000 deaths related to head trauma occur in US children annually.

Male gynecomastia

The mean age of onset for gynecomastia is 13 years and 2 months, with peak prevalence at 14 years of age. It is uncommon after age 17.

What immune cell is deficient when you see: recurrent oral ulcers, repeated invasive bacterial illnesses, and poor growth?

The most common disorder of neutrophil function is chronic granulomatous disease (CGD), a defect caused by a failure to produce oxygen radicals needed to kill phagocytosed microorganisms.

Gastroschisis facts

The pathogenesis of gastroschisis is unclear; it has been associated with low maternal age, low maternal body mass index, illicit drug use, cigarette smoking, and over-the-counter analgesia use. Necrotizing enterocolitis is seen in 15% to 20% of neonates with gastroschisis after surgical repair.

Rotavirus

There are age limits beyond which the vaccine should not be administered (14 weeks, 6 days for a first dose, 8 months, 0 days for the second or third doses); thus, delaying the vaccine could result in missed opportunities for protection.

Key features for psoriatic JIA?

The patient has clinical and radiographic findings consistent with psoriatic juvenile idiopathic arthritis (psJIA), which, at onset, typically involves a single or less than a total of 5 joints. The small joints of the hands, wrist, knee, ankle, or hip are most often affected; involvement of the distal interphalangeal joint is highly suggestive of psoriatic arthritis. Dactylitis (swelling of a digit extending beyond the joint borders) may occur along the length of the digit (the so-called "sausage digit") or be accentuated around an interphalangeal joint; the 2nd toe and index finger are most often affected. It is not uncommon for arthritis to present prior to any skin changes, which may be delayed up to 5—10 years following onset of arthritis and other systemic signs. Periostitis is common on plain radiographs of the digits. Associated clinical manifestations of acute or chronic anterior uveitis are common in girls, especially with ANA positivity (30%—50% of patients). Regular ophthalmologic screening is essential to prevent visual loss, and standard juvenile idiopathic arthritis (JIA) uveitis screening guidelines apply. The other manifestations shown as distractors are not seen in psJIA. Rather, one may see arthritis of the spine and/or sacroiliac joints, enthesitis (inflammation at the insertion of ligaments and tendons into bone), nail disease (pitting and onycholysis), and failure to thrive; fever is distinctly uncommon.

Pharmacokinetics of Marijuana

The peak is 15 to 30 min after smoking and 2 to 3 hours after ingestion. Effects last up to 4 hours after smoking and up to 12 hours after ingestion.

What is the tx for RMSF?

The preferred treatment in children of all ages, including children younger than 8 years, is doxycycline. Patients should be treated for 3 days after fever resolves, usually between 5 to 10 days total.

What is the skin diagnostic criteria for NF?

The presence of 6 or more macules measuring more than 5 mm in a child is one of the diagnostic criteria for neurofibromatosis type 1 (NF1). Six or more CALMs measuring more than 5 mm in a prepubertal individual or more than 1.5 cm in a postpubertal individual

EBV diagnosis

The presence of IgM antibody against viral capsid antigen in the absence of antibodies against Epstein-Barr nuclear antigen confirms the diagnosis of acute Epstein-Barr virus infection.

What is the recc daily intake of fiber?

The recommended daily intake is 5 + age (in years) grams.

What is the recurrence risk of NTD?

The recurrence risk in this situation for a future pregnancy with 1 sibling affected and neither parent affected is closest to 5%. If 2 siblings were affected, the recurrence risk would be higher, approaching 10% to 12%

Pediatric Emergency Care Applied Research Network (PECARN). What are the validated rules?

The risk of clinically important traumatic brain injury is estimated at more than 4% for children with a Glasgow coma scale score of 14, other signs of altered mental status, palpable skull fracture, or signs of basilar skull fracture.

What are associations with early pubertal changes?

The timing of the physical changes of puberty can have an impact on the development of a sense of self. Early pubertal changes in girls have been associated with lower self-esteem and poor body image.

What is the treatment of symptomatic Chiari 1 malformation?

The treatment for symptomatic Chiari I malformation is surgical decompression.

Neural tube defect lab stigmata?

The unborn child in the vignette likely has an open neural tube defect. Maternal quadruple serum screening in this clinical situation would demonstrate an elevated maternal serum a-fetoprotein (AFP) with normal human chorionic gonadotropin (hCG), inhibin A, and unconjugated estriol levels (μE3).

What is the treatment for congenital talipes equinovarus (TEV)

The widely accepted treatment for TEV is known as the Ponseti method. Treatment involves 3 stages: serial long leg casting, surgery to cut the Achilles tendon (for most infants), and long-term bracing to maintain the corrected position.

If a mother has PKU, what the increased risks for the baby if she is NOT treated?

There is an inverse relationship between cognitive function and maternal phenylalanine levels greater than 360 µmol/L during the pregnancy of a woman with phenylketonuria (PKU) (phenylalanine hydroxylase deficiency) that is poorly controlled.

What are the findings of congenital torticollis?

There may be skull and facial asymmetry, including jaw asymmetry, and the ear and eye ipsilateral to the injured side can appear smaller. The contralateral occipital and ipsilateral frontal areas are flattened.

How do you tell the difference between Metatarsus Abductus & congenital talipes equinovarus (TEV)

This diagnosis can be distinguished from TEV on physical examination; infants with metatarsus adductus have normal ankle motion, while those with TEV are fixed in a plantarflexed position.

How do you distinguish TAR vs Fanconi Anemia

Thrombocytopenia with absent radius presents with bilateral absent radii with shortened forearms with flexion at the elbow, resulting in an appearance of clubbed arms, but with sparing of the thumb. The normal thumb helps to distinguish this disorder from Fanconi anemia.

What is the most appropriate technique for blood pressure?

To obtain a meaningful blood pressure reading, the guidelines for proper measurement must be followed: use a cuff bladder width that is 40% of arm circumference; place cuff midway between olecranon and acromion; inflate the cuff to 20 mm Hg above the point at which the pulse is no longer palpated; and deflate the cuff no faster than 2 to 3 mm Hg per second.

Prune Belly Syndrome (Eagle-Barrett Syndrome)

Triad of b/l hydroureteral nephrosis, absence of abdominal wall musculature and undescended testes

Complication of twin to twin transfusion syndrome?

Twin-to-twin transfusion syndrome (TTTS) occurs when a vascular anastomosis develops between the placentas of the 2 fetuses. Hydrops fetalis can occur in both children. In the donor child, this is due to the associated anemia; in the recipient child, it is associated with polycythemia and the resulting hyperviscosity and fluid overload from the infused blood. Monochorionic, diamniotic placentation or monochorionic, monoamniotic placentation are at risk for TTTS.

coarsening of the facies, progressive skeletal dysplasia, macroglossia, and corneal clouding, in addition to neurodegenerative progression.

Type I MPS: Hurler syndrome

Patient has unprotected post-coital 4 days ago. What is most effective emergency contraception 72-120 hours after?

Ulipristal acetate (effective up to 5 days). Up to 72 hours you could use ethinyl estradiol + levonorgestrol or just levonorgesterol

When do you use ultrasound vs x-ray for developmental dysplasia of the hip?

Ultrasonography is the preferred imaging method to evaluate the hip in young infants before ossification of the femoral head, which occurs at about 5 or 6 months of age. Ultrasonographic screening for dysplasia before the age of 6 weeks results in a high rate of false-positive results.

What are the Ottawa ankle rules?

Unable to bear weight on the affected ankle or those with tenderness involving the tip or posterior aspect of the ankle malleoli.

How do you tell the difference between vocal cord paralysis and PVFD?

Unilateral vocal cord paralysis suggests nerve dysfunction and most often occurs after cardiac or neck surgery. In this patient, the lack of a weakened voice or chronic stridor makes a paralyzed vocal cord unlikely.

How does the arthritis between JIA and RA differ?

Unlike juvenile idiopathic arthritis (JIA), the arthritis associated with SLE is nonerosive and nondeforming.

Human Metapneumovirus

Unlike respiratory syncytial virus (RSV), human metapneumovirus (hMPV) disease can occur year round, although it is more common in the late winter or spring. Human metapneumovirus bronchiolitis is clinically indistinguishable from RSV bronchiolitis. Otitis media is common in children with hMPV respiratory infection.

A patient with suprapubic mass, has high Cr and electrolyte abnormalities (high PO4, high K). What is the next best step in management?

Urethral catheterization

Formulate a differential diagnosis of urinary tract obstruction Understand the various causes of urinary tract obstruction

Urinary flow obstruction in both kidneys, or a single functioning kidney, usually presents with acute renal failure. Bladder obstruction in children is most often observed in patients with abdominal soft tissue sarcomas or posterior urethral valves. Soft tissue sarcomas are rare in children. Rhabdomyosarcoma is the most common soft tissue sarcoma of childhood. PUV males born to mothers who received little or no prenatal care may present later with urinary tract infection, failure to thrive, abdominal distension (from an enlarged bladder), and a poor urinary stream or voiding dysfunction (urinary frequency, daytime and nocturnal enuresis, and poor urinary stream).

What is as sensitive as a 24-hour urine collection?

Urine protein to Cr ratio in first morning sample (any sample hiher than 0.2 is considered pathological).

How do you interpret urine sodium?

Urine sodium is therefore an indicator of intravascular volume status, and low urine sodium (< 20-25 mEq/L [20-25 mmol/L]) suggests decreased perfusion, even in patients without clinical features of dehydration.

What is the use of nasal nitric oxide?

Use of nasal nitric oxide measurements is emerging as a potentially sensitive and noninvasive tool for the diagnosis of PCD in patients with suggestive clinical findings. At present, nasal nitric oxide remains a research tool.

Diphenylcyclopropenone [DPCP]

Used for alopecia areata - changes immune system from hair loss to contact dermatitis

Varicella post-exposure prophylaxis

Varicella vaccine is recommended within 3 to 5 days of exposure for healthy VZV-nonimmune individuals older than 12 months. Individuals with a history of only 1 prior dose of vaccine should receive the second dose if 3 months have elapsed since the first dose.

When should children have varicella VIG?

Varicella-zoster immune globulin for postexposure prophylaxis is recommended for the following high-risk individuals who have significant exposure to varicella and who cannot be immunized: •Neonates born to mothers who develop chickenpox within 5 days before and 48 hours after delivery •Neonates at greater than 28 weeks' gestation whose mothers lack evidence of immunity •Neonates at less than 28 weeks' gestation or weighing less than 1000 g in birth weight regardless of maternal immunity •Pregnant women without evidence of immunity •Immunocompromised persons without evidence of immunity

Understand the potential nutritional deficiencies in adolescents

Vegan diets pose potentially more serious health risks (deficiency of vitamin B12 but to a lesser extent vitamins A and D and calcium, iron, and zinc) compared with vegetarian diets.

What is the most common cause of pneumonia in children 3 months to 4 years of age?

Viral

16 y/o girl became vegan 6 mo. ago. Menses are reg. What is her GREATEST risk of deficiency?

Vitamin B12.

Review the Jones Criteria for ARF.

We need a confirmation of GAS infection. Positive rapid antigen test (throat cx neg during ARF at times) Rising ASO titer. AND 1 major 2 minor or 2 major

What is the genetic inheritance pattern for CGD?

both X-linked and autosomal recessive forms. The X-linked form is by far the most common and is caused by a mutation in the CYBB gene that results in a dysfunctional gp91 protein. The most common autosomal form of CGD is caused by a mutation in the NCF1 gene on chromosome 7, resulting in a dysfunctional p47 protein

At the initiation of the growth spurt, boys are on average how many cm taller than girls?

When compared with girls, boys experience an additional 2 years of growth (at a rate of about 5 cm/year) before growth spurt initiation As a result, girls are about 10 cm shorter than boys at IGS, which is the primary explanation for the observed 13-cm mean difference in heights of adult women and men.

A full-term newborn was delivered SVAD. Findings on physical examination are unremarkable except for bilateral ear pits and a small branchial sinus on the left neck with no drainage noted. Dx?

branchio-oto-renal (BOR) syndrome. This autosomal dominant disorder caused by mutations in EYA1

congenital varicella syndrome

a. Limb hypoplasia b. Cutanous scarring in dermatonal distribution c. Blindness (microphthalmia)

What is the likely organism involving piercing of the helix of the ear?

While Staphylococcus aureus remains an important cause of perichondritis, Pseudomonas aeruginosa

What are the guidelines for a baby born with stigmata of BWS

While screening recommendations vary slightly, surveillance guidelines published in 2017 recommend measuring serum α-fetoprotein every 3 months for the first 4 years to screen for hepatoblastoma and complete abdominal ultrasonography every 3 months for the first 7 years to screen for Wilms tumor.

What is the observation period for PECARN?

While there is no definite consensus regarding the optimal observation period for children following minor closed head injury, some experts have recommended an observation period of 4 to 6 hours.

What is the time frame to give varicella Ig?

While varicella-zoster immune globulin should be given as soon as possible after exposure, limited data suggest there may be efficacy up to 10 days after exposure. Based on expert opinion, intravenous immune globulin can be used for candidates as passive immunoprophylaxis if the varicella-specific formulation cannot be obtained.

Congenital syphilis

Widespread periostal elevations Pseudoparalysis of infancy

Key principles of green-stick fractures

With greenstick injuries, the affected long bone bends before it breaks, with the thick and active periosteum remaining intact on 1 side. Greenstick fractures with angulation of more than 15 degrees require closed reduction, immobilization in a splint, and orthopedic follow-up. To restore anatomic alignment, the fracture often must be completed.

_____________________________, also known as lysosomal acid lipase deficiency, presents with infantile-onset malabsorption because of the storage of cholesterol esters and triglycerides in hepatic macrophages that leads to hepatomegaly, liver disease, and adrenal gland calcification/adrenal cortical insufficiency

Wolman Disease

What are the guidelines for MSM and screening?

Young men who have sex with men: Annual Screening for human immunodeficiency virus and syphilis Annual screening for C trachomatis and N gonorrhoeaIf practicing insertive anal intercourse: urine NAAT for both organisms If practicing receptive anal intercourse: rectal swab NAAT for both organisms If practicing receptive oral intercourse: pharyngeal swab NAAT for N gonorrhoeae only (testing for pharyngeal infection with C trachomatis is not recommended)

When would a observational study be needede in conjunction with RCT?

a well-constructed observational study may be needed if the use of a placebo when the alternative treatment option is generally thought to be of benefit or if a placebo would deprive the patient of the usual standard medical care.

What are symptoms of spasmus nutans?

an idiopathic condition of infancy, is characterized by horizontal nystagmus, head tilting, and head nodding.

SMA syndrome will have duodenal obstruction in distal portions and what other things on history?

anorexia, bedrest, body casting, abdominal surgery complaints usually when lying prone or with knees to chest

What is Benign paroxysmal torticollis of infancy (BPTI)

another cause of head tilt, which presents in the first several months after birth. Symptoms may also include vomiting, irritability, and increased drowsiness.

When do you screen early for lipid?

be age 55 years or younger for a man and age 65 years or younger for a woman Otherwise 9-11 and 17 to 19

Latrodectus mactans bite

black widow Clinical manifestations resulting from black widow spider bites typically include generalized muscle pain and rigidity, developing 1 to 8 hours after the bite. Anti venom needed

What is the presentation of A vein of Galen aneurysmal malformation

can present in the neonatal period or in infancy with congestive heart failure because of the large blood volume that is shunted directly back into the right heart.

Maternal diabetes has what heart and CNS changes in new-borns?

congenital heart disease (transposition of the great vessels, ventricular septal defect), caudal regression syndrome, and neural tube defects (anencephaly, spina bifida).

Shprintzen-Goldberg Syndrome

coronal, sagittal, or lambdoid craniosynostosis; dolichocephaly; marfanoid skeletal feature Brain anomalies include hydrocephalus, lateral ventricular dilatation, and Chiari 1 malformation. Shprintzen-Goldberg syndrome is inherited as an autosomal dominant disorder due to mutations in the SKI gene.

cholesteatoma

cystlike mass composed of epithelial cells and cholesterol occurring in the middle ear; may be associated with chronic otitis media surgical tx for management

Female growth spurt age:

growth spurt (IGS) typically occurs around age 9.5 years

Branchio-oto-renal syndrome (BOR)

deafness, external ear deformities, lateral semicircular canal hypoplasia, branchial arch anomalies, and renal malformations

What is the definition of Chiari 1

defined as the descent of the cerebellar tonsils at least 5 mm below the foramen magnum.

What are risk factors for children who will have speech or language delay from OME?

developmental delays, hearing loss independent of OME, syndromes associated with language delay, cleft palate, craniofacial anomalies, or visual impairment. These kids need audiometry

What are features of vasculitic urticaria?

duration of lesions >72 hours, pain, discoloration, or residual scarring

What is a "significant clinical finding"

dysmorphic facies, facial asymmetry, ocular or eyelid colobomas, choanal atresia, micrognathia (small jaw), branchial cysts or sinuses, cardiac murmur, limb anomalies, or an imperforate or anteriorly placed anus.

What are risk factors for developmental dysplasia of the hip?

female sex, firstborn status, breech presentation, family history, and oligohydramnios. Positional musculoskeletal deformities (eg, metatarsus adductus, congenital torticollis) may also be associated with DDH.

What are the minor criteria for Rheumatic fever?

fever, arthralgia, prolonged PR interval on electrocardiograph, and elevated acute-phase reactants. (1 major and 2 minor).

Cyclical vomiting syndrome

for a few hours to a day no other symptoms - dx of exclusion triggers similar to migraines no abdominal pain

What is the Galeazzi sign?

he knee and hip flexed, the thigh will appear shorter on the affected side because of the superior position of the femoral head.

tibial torsion

inward twisting of tibia

What is the The CRAFFT tool

is a mnemonic acronym of the first letters of key words in the screening questions for alcoholism: car, relax, alone, friends (or family), forgetting, and trouble. Endorsement of 2 or more CRAFFT behaviors should prompt further assessment.

Treacher Collins syndrome

mandibular and zygomatic hypoplasia, coloboma of the lower eyelids, absent lower eyelashes, external ear abnormalities, and preauricular hair displacement onto the cheekbones. Diagnosis is made via clinical and radiographic findings. Genetic testing is available for the 3 genes known to cause this disorder: TCOF1, POLR1C, and POLRID.

What are the major criteria for Rheumatic fever?

polyarthritis, carditis, Sydenham chorea, subcutaneous nodules, and erythema marginatum. (2 major)

What is the treatment of allergic contact dermatitis?

topical corticosteroids. Treatment consists of cool compresses, as well as medium-to-high dose topical corticosteroids. An oral antihistamine is often also used to decrease the pruritus.

What is the mechanism of a greenstick fracture?

typical mechanism of a greenstick fracture is a fall onto an outstretched hand.

4 yo boy who is on conslidation chemotherapy who presents after varicella exposure. What is the next best step in management?

varicella-zoster immune globulin Passive immunoprophylaxis after exposure to varicella is indicated in individuals likely to develop infection if exposed and likely to have complications if they develop infection.

Toxic shock syndrome criteria

• Fever > to 38.9°C (102°F)• Rash (typically diffuse erythroderma)• Desquamation (commonly palms/soles 1 to 2 weeks after the onset of symptoms) • Hypotension (systolic blood pressure less than fifth percentile for age for children younger than 16 years of age, ≤ 90 mm Hg for ≥ 16 years of age)• Multisystem involvement (in 3 or more organ symptoms): Gastrointestinal - vomiting or diarrhea at onset of illness Musculoskeletal - severe myalgias at onset of illness or creatine phosphokinase (CPK) greater than twice the upper limit of normal Mucocutaneous - vaginal, oropharyngeal, and/or conjunctival hyperemia Renal - blood urea nitrogen (BUN) or creatinine greater than twice the upper limit of normal, or urine with greater than 5 white blood cells/high power field without a urinary tract infection Hepatic - total bilirubin or aspartate aminotransferase/alanine aminotransferase greater than twice the upper limit of normal Hematologic - platelet count less than 100 x 103/μL (100 x 109/L) Central nervous system (CNS) - altered mental status without focal neurologic signs when afebrile and normotensive

What is the management (long term) of HSP?

•Patients with HSP should be monitored for the occurrence of renal disease for 6 months after diagnosis.

What ID studies should be placed on internationally adopted children?

◦Hepatitis B ◦Hepatitis C ◦Syphilis ◦Human immunodeficiency virus ◦Giardia intestinalis ◦Cryptosporidium species ◦Tuberculosis ◦Trypanosoma cruzi (if from endemic area) ◦Lymphatic filariasis (if from endemic area) ◦Strongyloides species (if eosinophilia present) ◦Schistosoma species (if eosinophilia present and from endemic area)


Related study sets

MBIO 4310 - Final Exam Review Questions

View Set

Art Appreciation: Unit 3: Quizzes

View Set

PrepU Adult Health: Practice Exam 1

View Set